Menu Close

Category: Logarithms

Question-153959

Question Number 153959 by DELETED last updated on 12/Sep/21 Answered by DELETED last updated on 12/Sep/21 $$\mathrm{F}_{\mathrm{AC}} =\mathrm{k}.\frac{\mathrm{q}_{\mathrm{a}} .\mathrm{q}_{\mathrm{c}} }{\mathrm{r}_{\mathrm{AC}} ^{\mathrm{2}} }\: \\ $$$$\:\:\:\:\:\:\:\:=\mathrm{9}.\mathrm{10}^{\mathrm{9}} .\frac{\mathrm{2}.\mathrm{10}^{−\mathrm{6}}…

Question-153950

Question Number 153950 by liberty last updated on 12/Sep/21 Answered by EDWIN88 last updated on 13/Sep/21 $$\:\:\mathrm{log}\:_{\mid{x}−\frac{\mathrm{7}}{\mathrm{4}}\mid} \left(\mathrm{log}\:_{\frac{\mathrm{1}}{\mathrm{2}}} {x}\right)\leqslant\:\mathrm{0} \\ $$$$\Rightarrow\mathrm{log}\:_{\mid{x}−\frac{\mathrm{7}}{\mathrm{4}}\mid} \left(\mathrm{log}\:_{\frac{\mathrm{1}}{\mathrm{2}}} {x}\right)\:\leqslant\:\mathrm{log}\:_{\mid{x}−\frac{\mathrm{7}}{\mathrm{4}}\mid} \left(\mathrm{1}\right) \\…

Question-88068

Question Number 88068 by student work last updated on 08/Apr/20 Commented by john santu last updated on 08/Apr/20 $${do}\:{you}\:{mean}\:\mathrm{ln}\:{x}^{\mathrm{2}} \:.\mathrm{2log}_{\mathrm{2}{x}} \:\left({x}\right)\:=\:\mathrm{log}_{\mathrm{4}{x}} \left(\mathrm{2}\right)?? \\ $$ Commented…

x-3-x-1-1-4-x-3-x-2-1-3-

Question Number 87637 by john santu last updated on 05/Apr/20 $$\sqrt[{\mathrm{4}\:\:}]{\mid\mathrm{x}−\mathrm{3}\mid^{\mathrm{x}+\mathrm{1}} }\:=\:\sqrt[{\mathrm{3}\:\:}]{\mid\mathrm{x}−\mathrm{3}\mid^{\mathrm{x}−\mathrm{2}} } \\ $$ Answered by TANMAY PANACEA. last updated on 05/Apr/20 $$\mid{x}−\mathrm{3}\mid^{\frac{{x}+\mathrm{1}}{\mathrm{4}}} =\mid{x}−\mathrm{3}\mid^{\frac{{x}−\mathrm{2}}{\mathrm{3}}}…

dx-x-2-2x-2-x-2-2x-4-

Question Number 153109 by peter frank last updated on 04/Sep/21 $$\int\frac{\mathrm{dx}}{\mathrm{x}^{\mathrm{2}} +\mathrm{2x}+\mathrm{2}\sqrt{\mathrm{x}^{\mathrm{2}} +\mathrm{2x}−\mathrm{4}}} \\ $$ Commented by MJS_new last updated on 05/Sep/21 $$\mathrm{the}\:\mathrm{path}\:\mathrm{is}\:\mathrm{clear}\:\mathrm{but}\:\mathrm{the}\:\mathrm{constants}\:\mathrm{are}\:\mathrm{weird} \\ $$$${t}=\frac{{x}+\mathrm{1}+\sqrt{{x}^{\mathrm{2}}…